LSAT and Law School Admissions Forum

Get expert LSAT preparation and law school admissions advice from PowerScore Test Preparation.

 Administrator
PowerScore Staff
  • PowerScore Staff
  • Posts: 8917
  • Joined: Feb 02, 2011
|
#40320
Complete Question Explanation
(The complete setup for this game can be found here: lsat/viewtopic.php?t=6748)

The correct answer choice is (B)

This question asks us to identify how many reports could be first in the first segment, assuming that N is the first report in the second segment. The Not Laws deduced during our setup will prove exceptionally useful here. As we know, the first report in the first segment cannot be S or W. This is because S can never be the longest report in either segment, and W cannot be the longest report in the first segment, for the reasons mentioned earlier. Therefore, there are only two reports, either one of which could be first in the first segment. Answer choice (B) is therefore correct.

Get the most out of your LSAT Prep Plus subscription.

Analyze and track your performance with our Testing and Analytics Package.